Skip to Main Content

PrepTest 79, Game 2, Question 11

Transcript

If L and O are assigned to the same area, then which one of the following could be true? So we're giving some stipulations, we're trying to get L and O together in the same area, and then figure out, based on that, what could be true? Now, if O and L are going together, they are not going together in area 1, and that's because O can't go in area 1.

So O and L can't hang out together there. Now, we can figure out the same thing about area 2 as well. Why is that so? So if O and L are together in area 2, that means that L is not in area 3 pairing with M. So that means that L, while hanging out with O in area 2, is going to need to pair with K, but we can't put K here.

So when we put O in area 2, J and K need to be in the same area. It doesn't need to be in the same area as O, but they need to be in the same area as each other. Now, there's no way, there is not enough space here to fit O, L, J, and K, but we need at least K here, but we can't separate those two. So O and L going in 2 is gonna create a create a problem, meeting the requirements of this rule and this rule at the same time.

So O and L cannot go together in 1, and O and L cannot go together in 2. So we're gonna end up with O and L only being able to go together in 3. So we figured out area 3 in its entirety. We're gonna have M, O and L. Based on this, we wanna figure out what could be true. So the correct answer is B.

K is assigned to area 2. So we know we have M, O, L here. If we put K in area 2, that's cool. We also know that J and K cannot be assigned to the same area since O is not in 2. So that's gonna force J up here into area 1.

P cannot go into area 1, so that's gonna force it into area 2. This could be true, so this is our correct answer choice.

Read full transcript